Merge remote branch 'public/master'
[course.git] / latex / problems / Serway_and_Jewett_4 / problem04.08.tex
1 \begin{problem*}{4.8}
2 Three forces, given by 
3  $\vect{F}_1 = (-2.00\vect{i} + 2.00\vect{j})\U{N}$,
4  $\vect{F}_2 = ( 5.00\vect{i} - 3.00\vect{j})\U{N}$, and 
5  $\vect{F}_3 =  -45.0\vect{i}\U{N}$,
6  act on an object to give it an acceleration of magnitude
7  $a = 3.75\U{m/s}^2$
8  \Part{a} What is the direction of the acceleration?
9  \Part{b} What is the mass of the object?
10  \Part{c} If the object is initially at rest, what is its speed $v$ 
11   after $t = 10.0\U{s}$?
12  \Part{d} What are the velocity components of the object after
13   $t = 10.0\U{s}$?
14 \end{problem*} % problem 4.8
15
16 \begin{solution}
17 \Part{a}
18 Summing the forces we have
19 \begin{align}
20  \sum F_x &= F_1x + F_2x + F_3x = (-2.00 + 5.00 - 45.0)\U{N} = -42.0\U{N} \\
21  \sum F_y &= F_1y + F_2y + F_3y = (+2.00 - 3.00 + 0)\U{N} = -1.00\U{N}
22 \end{align}
23 We know from Newtons second law that
24 \begin{equation}
25  \sum \vect{F} = m \vect{a}
26 \end{equation}
27 So the acceleration $\vect{a}$ will be in the same direction as the 
28 force $\vect{F}$.
29 The direction $\theta$ of the force is given by
30 \begin{equation}
31  \theta = \arctan \left( \frac{F_y}{F_x} \right)
32         = \arctan \left( \frac{-1}{-42} \right)
33         = (1.36 + 180)^o = \ans{181.36^o}
34 \end{equation}
35 Measured counter-clockwise from the $\vect{x}$ axis
36  (where we have added $180^o$ because $F_x < 0$ so we have a backside 
37   $\arctan$).
38
39 \Part{b}
40 From Newton's second law
41 \begin{align}
42  \sum \vect{F} &= m \vect{a} \\
43  \left|\sum \vect{F}\right| &= m \left|\vect{a}\right| \\
44  m &= \frac{\left|\sum \vect{F}\right|}{a}
45     = \frac{ \sqrt{(-41.0)^2 + (-1.00)^2}\U{N}}{3.75\U{m/s}^2}
46     = \ans{11.2\U{kg}}
47 \end{align}
48
49 \Part{c}
50 This section is constant acceleration review.
51 \begin{equation}
52  v = a t + v_0
53    = 3.75\U{m/s}^2 \cdot 10.0\U{s} = \ans{37.5\U{m/s}}
54 \end{equation}
55
56 \Part{d}
57 Using our velocity $v = |\vect{v}|$ from \Part{c} and our angle
58  $\theta$ from \Part{a} (we know that $\vect{v}$ is in the same 
59 direction as $\vect{a}$ and $\vect{F}$) we have
60 \begin{align}
61  v_x &= v \cos \theta = 37.5\U{m/s} \cdot \cos 181.36^o
62       = -37.49\U{m/s} \\
63  v_y &= v \sin \theta = 37.5\U{m/s} \cdot \sin 181.36^o
64       = -0.893\U{m/s} \\
65  \vect{v} &= \ans{(-37.49\vect{i} - 0.893\vect{j})\U{m/s}}
66 \end{align}
67 \end{solution}